Comparison of Bounded Operator Topologies












3














I'm looking at the topologies on the set of bounded operators between Banach spaces, $L(X,Y)$, and while I see how uniform convergence implies strong convergence implies weak convergence, I'm struggling to see why we have that the weak operator topology is weaker than the strong operator topology which in turn is weaker than the uniform operator topology:
$$
tau_{rm weak}subset tau_{rm strong}subsettau_{rm uniform}
$$










share|cite|improve this question



























    3














    I'm looking at the topologies on the set of bounded operators between Banach spaces, $L(X,Y)$, and while I see how uniform convergence implies strong convergence implies weak convergence, I'm struggling to see why we have that the weak operator topology is weaker than the strong operator topology which in turn is weaker than the uniform operator topology:
    $$
    tau_{rm weak}subset tau_{rm strong}subsettau_{rm uniform}
    $$










    share|cite|improve this question

























      3












      3








      3







      I'm looking at the topologies on the set of bounded operators between Banach spaces, $L(X,Y)$, and while I see how uniform convergence implies strong convergence implies weak convergence, I'm struggling to see why we have that the weak operator topology is weaker than the strong operator topology which in turn is weaker than the uniform operator topology:
      $$
      tau_{rm weak}subset tau_{rm strong}subsettau_{rm uniform}
      $$










      share|cite|improve this question













      I'm looking at the topologies on the set of bounded operators between Banach spaces, $L(X,Y)$, and while I see how uniform convergence implies strong convergence implies weak convergence, I'm struggling to see why we have that the weak operator topology is weaker than the strong operator topology which in turn is weaker than the uniform operator topology:
      $$
      tau_{rm weak}subset tau_{rm strong}subsettau_{rm uniform}
      $$







      general-topology functional-analysis






      share|cite|improve this question













      share|cite|improve this question











      share|cite|improve this question




      share|cite|improve this question










      asked Nov 20 at 1:14









      user2379888

      23719




      23719






















          1 Answer
          1






          active

          oldest

          votes


















          0














          The best case to see why these topologies differ is in Hilbert space. Consider the case that $X=Y=ell^2$, and let ${e_n}_{ninmathbb N}$ be an orthonormal basis for $ell^2$.



          Let $P_nin L(ell^2)$ be the orthogonal projection onto $operatorname{span}{e_1,ldots,e_n}$. Then ${P_n}$ converges strongly to the identity operator $I$, but $|P_n-I|=1$ for all $n$. This shows that the strong operator topology is strictly weaker than the norm topology.



          To show that the weak operator topology is strictly weaker than the strong operator topology, it suffices to show that the adjoint operation $Tmapsto T^*$ is continuous in the weak operator topology but not in the strong operator topology.



          Showing that the adjoint operation is continuous in the weak operator topology is very simple (just take a weakly converging net, and show the adjoint converges weakly by rearranging). To show that this is not continuous in the strong operator topology, consider the operators $T_n:ell^2toell^2$ defined by $T_n(x)=langle x,e_nrangle e_1.$ Then $|T_n(x)|=|langle x,e_nrangle|to 0$ for any $xinell^2$, so ${T_n}$ converges strongly to $0$. But $T_n^*(x)=langle x,e_1rangle e_n$, and $|T_n^*(e_1)|=1$ for all $n$, so $T_n^*$ does not converge strongly to $0$.






          share|cite|improve this answer





















            Your Answer





            StackExchange.ifUsing("editor", function () {
            return StackExchange.using("mathjaxEditing", function () {
            StackExchange.MarkdownEditor.creationCallbacks.add(function (editor, postfix) {
            StackExchange.mathjaxEditing.prepareWmdForMathJax(editor, postfix, [["$", "$"], ["\\(","\\)"]]);
            });
            });
            }, "mathjax-editing");

            StackExchange.ready(function() {
            var channelOptions = {
            tags: "".split(" "),
            id: "69"
            };
            initTagRenderer("".split(" "), "".split(" "), channelOptions);

            StackExchange.using("externalEditor", function() {
            // Have to fire editor after snippets, if snippets enabled
            if (StackExchange.settings.snippets.snippetsEnabled) {
            StackExchange.using("snippets", function() {
            createEditor();
            });
            }
            else {
            createEditor();
            }
            });

            function createEditor() {
            StackExchange.prepareEditor({
            heartbeatType: 'answer',
            autoActivateHeartbeat: false,
            convertImagesToLinks: true,
            noModals: true,
            showLowRepImageUploadWarning: true,
            reputationToPostImages: 10,
            bindNavPrevention: true,
            postfix: "",
            imageUploader: {
            brandingHtml: "Powered by u003ca class="icon-imgur-white" href="https://imgur.com/"u003eu003c/au003e",
            contentPolicyHtml: "User contributions licensed under u003ca href="https://creativecommons.org/licenses/by-sa/3.0/"u003ecc by-sa 3.0 with attribution requiredu003c/au003e u003ca href="https://stackoverflow.com/legal/content-policy"u003e(content policy)u003c/au003e",
            allowUrls: true
            },
            noCode: true, onDemand: true,
            discardSelector: ".discard-answer"
            ,immediatelyShowMarkdownHelp:true
            });


            }
            });














            draft saved

            draft discarded


















            StackExchange.ready(
            function () {
            StackExchange.openid.initPostLogin('.new-post-login', 'https%3a%2f%2fmath.stackexchange.com%2fquestions%2f3005795%2fcomparison-of-bounded-operator-topologies%23new-answer', 'question_page');
            }
            );

            Post as a guest















            Required, but never shown

























            1 Answer
            1






            active

            oldest

            votes








            1 Answer
            1






            active

            oldest

            votes









            active

            oldest

            votes






            active

            oldest

            votes









            0














            The best case to see why these topologies differ is in Hilbert space. Consider the case that $X=Y=ell^2$, and let ${e_n}_{ninmathbb N}$ be an orthonormal basis for $ell^2$.



            Let $P_nin L(ell^2)$ be the orthogonal projection onto $operatorname{span}{e_1,ldots,e_n}$. Then ${P_n}$ converges strongly to the identity operator $I$, but $|P_n-I|=1$ for all $n$. This shows that the strong operator topology is strictly weaker than the norm topology.



            To show that the weak operator topology is strictly weaker than the strong operator topology, it suffices to show that the adjoint operation $Tmapsto T^*$ is continuous in the weak operator topology but not in the strong operator topology.



            Showing that the adjoint operation is continuous in the weak operator topology is very simple (just take a weakly converging net, and show the adjoint converges weakly by rearranging). To show that this is not continuous in the strong operator topology, consider the operators $T_n:ell^2toell^2$ defined by $T_n(x)=langle x,e_nrangle e_1.$ Then $|T_n(x)|=|langle x,e_nrangle|to 0$ for any $xinell^2$, so ${T_n}$ converges strongly to $0$. But $T_n^*(x)=langle x,e_1rangle e_n$, and $|T_n^*(e_1)|=1$ for all $n$, so $T_n^*$ does not converge strongly to $0$.






            share|cite|improve this answer


























              0














              The best case to see why these topologies differ is in Hilbert space. Consider the case that $X=Y=ell^2$, and let ${e_n}_{ninmathbb N}$ be an orthonormal basis for $ell^2$.



              Let $P_nin L(ell^2)$ be the orthogonal projection onto $operatorname{span}{e_1,ldots,e_n}$. Then ${P_n}$ converges strongly to the identity operator $I$, but $|P_n-I|=1$ for all $n$. This shows that the strong operator topology is strictly weaker than the norm topology.



              To show that the weak operator topology is strictly weaker than the strong operator topology, it suffices to show that the adjoint operation $Tmapsto T^*$ is continuous in the weak operator topology but not in the strong operator topology.



              Showing that the adjoint operation is continuous in the weak operator topology is very simple (just take a weakly converging net, and show the adjoint converges weakly by rearranging). To show that this is not continuous in the strong operator topology, consider the operators $T_n:ell^2toell^2$ defined by $T_n(x)=langle x,e_nrangle e_1.$ Then $|T_n(x)|=|langle x,e_nrangle|to 0$ for any $xinell^2$, so ${T_n}$ converges strongly to $0$. But $T_n^*(x)=langle x,e_1rangle e_n$, and $|T_n^*(e_1)|=1$ for all $n$, so $T_n^*$ does not converge strongly to $0$.






              share|cite|improve this answer
























                0












                0








                0






                The best case to see why these topologies differ is in Hilbert space. Consider the case that $X=Y=ell^2$, and let ${e_n}_{ninmathbb N}$ be an orthonormal basis for $ell^2$.



                Let $P_nin L(ell^2)$ be the orthogonal projection onto $operatorname{span}{e_1,ldots,e_n}$. Then ${P_n}$ converges strongly to the identity operator $I$, but $|P_n-I|=1$ for all $n$. This shows that the strong operator topology is strictly weaker than the norm topology.



                To show that the weak operator topology is strictly weaker than the strong operator topology, it suffices to show that the adjoint operation $Tmapsto T^*$ is continuous in the weak operator topology but not in the strong operator topology.



                Showing that the adjoint operation is continuous in the weak operator topology is very simple (just take a weakly converging net, and show the adjoint converges weakly by rearranging). To show that this is not continuous in the strong operator topology, consider the operators $T_n:ell^2toell^2$ defined by $T_n(x)=langle x,e_nrangle e_1.$ Then $|T_n(x)|=|langle x,e_nrangle|to 0$ for any $xinell^2$, so ${T_n}$ converges strongly to $0$. But $T_n^*(x)=langle x,e_1rangle e_n$, and $|T_n^*(e_1)|=1$ for all $n$, so $T_n^*$ does not converge strongly to $0$.






                share|cite|improve this answer












                The best case to see why these topologies differ is in Hilbert space. Consider the case that $X=Y=ell^2$, and let ${e_n}_{ninmathbb N}$ be an orthonormal basis for $ell^2$.



                Let $P_nin L(ell^2)$ be the orthogonal projection onto $operatorname{span}{e_1,ldots,e_n}$. Then ${P_n}$ converges strongly to the identity operator $I$, but $|P_n-I|=1$ for all $n$. This shows that the strong operator topology is strictly weaker than the norm topology.



                To show that the weak operator topology is strictly weaker than the strong operator topology, it suffices to show that the adjoint operation $Tmapsto T^*$ is continuous in the weak operator topology but not in the strong operator topology.



                Showing that the adjoint operation is continuous in the weak operator topology is very simple (just take a weakly converging net, and show the adjoint converges weakly by rearranging). To show that this is not continuous in the strong operator topology, consider the operators $T_n:ell^2toell^2$ defined by $T_n(x)=langle x,e_nrangle e_1.$ Then $|T_n(x)|=|langle x,e_nrangle|to 0$ for any $xinell^2$, so ${T_n}$ converges strongly to $0$. But $T_n^*(x)=langle x,e_1rangle e_n$, and $|T_n^*(e_1)|=1$ for all $n$, so $T_n^*$ does not converge strongly to $0$.







                share|cite|improve this answer












                share|cite|improve this answer



                share|cite|improve this answer










                answered Nov 20 at 2:44









                Aweygan

                13.4k21441




                13.4k21441






























                    draft saved

                    draft discarded




















































                    Thanks for contributing an answer to Mathematics Stack Exchange!


                    • Please be sure to answer the question. Provide details and share your research!

                    But avoid



                    • Asking for help, clarification, or responding to other answers.

                    • Making statements based on opinion; back them up with references or personal experience.


                    Use MathJax to format equations. MathJax reference.


                    To learn more, see our tips on writing great answers.





                    Some of your past answers have not been well-received, and you're in danger of being blocked from answering.


                    Please pay close attention to the following guidance:


                    • Please be sure to answer the question. Provide details and share your research!

                    But avoid



                    • Asking for help, clarification, or responding to other answers.

                    • Making statements based on opinion; back them up with references or personal experience.


                    To learn more, see our tips on writing great answers.




                    draft saved


                    draft discarded














                    StackExchange.ready(
                    function () {
                    StackExchange.openid.initPostLogin('.new-post-login', 'https%3a%2f%2fmath.stackexchange.com%2fquestions%2f3005795%2fcomparison-of-bounded-operator-topologies%23new-answer', 'question_page');
                    }
                    );

                    Post as a guest















                    Required, but never shown





















































                    Required, but never shown














                    Required, but never shown












                    Required, but never shown







                    Required, but never shown

































                    Required, but never shown














                    Required, but never shown












                    Required, but never shown







                    Required, but never shown







                    Popular posts from this blog

                    Biblatex bibliography style without URLs when DOI exists (in Overleaf with Zotero bibliography)

                    ComboBox Display Member on multiple fields

                    Is it possible to collect Nectar points via Trainline?